Đến nội dung

mduc123

mduc123

Đăng ký: 13-04-2018
Offline Đăng nhập: 31-05-2018 - 14:04
***--

#709126 $u_{0}=\frac{1}{2},u_{k+1}=...

Gửi bởi mduc123 trong 23-05-2018 - 16:41

Cho dãy $(u_{n})$ thoả mãn: $\left\{\begin{matrix} u_{0}=\frac{1}{2}\\u_{k+1}=u_{k}+\frac{1}{n}u_{k}^{2},\forall k=\overline{0,n-1} \end{matrix}\right.$

Tìm $\lim u_{n}$

Lời giải:

$u_{k+1}=u_{k}+\frac{1}{n}u_{k}^{2}\Rightarrow nu_{k+1}=nu_{k}+u_{k}^{2}\Rightarrow (u_{k+1}-u_{k})(n+u_{k})=u_{k}u_{k+1}$

$\Rightarrow \frac{1}{u_{k}}-\frac{1}{u_{k+1}}=\frac{1}{n+u_{k}}$. 

$\Rightarrow \sum_{j=0}^{n-1}(\frac{1}{u_{j}}-\frac{1}{u_{j+1}})=\frac{1}{u_{0}}-\frac{1}{u_{n}}=2-\frac{1}{u_{n}}< n.\frac{1}{n}=1\Rightarrow u_{n}< 1$ (1)

$2-\frac{1}{u_{n}}> \frac{n}{n+1}$ (do $u_{n}<1$ ) $u_{n}> \frac{n+1}{n+2}$ (2)

Từ (1) và (2) $\frac{n+1}{n+2}< u_{n} < 1$.

Theo nguyên lý kẹp ta có $\lim_{n\rightarrow +\infty }u_{n}=1$




#709108 Đề chọn học sinh giỏi tỉnh Thanh Hóa

Gửi bởi mduc123 trong 23-05-2018 - 13:14

        SỞ GD&ĐT THANH HÓA                                                                                       KỲ THI CHỌN HỌC SINH GIỎI KHỐI 11

TRƯỜNG THPT CHUYÊN LAM SƠN                                                                                            NĂM HỌC 2018-2019

                                                                                                                                                               Môn thi:Toán

                                                                                                                                                     thời gian làm bài: 180 phút

 

 

Ngày 1:

Bài 1: Cho dãy số $(x_{n})$ thỏa mãn $\left\{\begin{matrix} x_{1}\in (1;2) & \\ x_{n+1}=1+x_{n}-\frac{x_{n}^{2}}{2} & (n\in N^{*}) \end{matrix}\right.$ 

Chứng minh dãy $(x_{n})$ có giới hạn và tìm giới hạn đó.

Bài 2: Cho đa thức P(x) có bậc n nguyên dương, $n\geq 2$. Biết phương trình P(x)=0 có n nghiệm thực $x_{1},x_{2},...,x_{n}$ phân biệt.

Chứng minh $\frac{1}{P'(x_{1})}+\frac{1}{P'(x_{2})}+...+\frac{1}{P'(x_{n})}=0$

Bài 3: Cho tam giác ABC nhọn có BC>CA. Gọi O,H lần lượt là tâm đường tròn ngoại tiếp, trực tâm của tam giác ABC; F là chân đường cao hạ từ C của tam giác ABC. Đường thẳng vuông góc với OF tại F cắt đường thẳng chứa cạnh AC tại P. Chừng minh $\widehat{FHP}=\widehat{BAC}$

Bài 4: Chứng minh rằng một hình vuông bất kỳ có thể cắt thành n hình vuông với mọi số n nguyên dương, $n\geq 6$. Chứng minh rằng điều này không thể thực hiện được với n=5

 

Ngày 2:

Bài 5: Tìm tất cả các hàm $f:\mathbb{R}\rightarrow \mathbb{R}$ thỏa mãn điều kiện:

$f(2x+f(y))=f(2x)+xf(2y)+f(f(y))$ với $\forall x,y\in \mathbb{R}$

Bài 6: Giả sử $p\geq 5$ là số nguyên tố. Chứng minh rằng tồn tại các số nguyên dương m,n sao cho $m+n\leq \frac{p-1}{2}$ và $(2^{n}3^{m}-1)\vdots p$

Bài 7: Các đỉnh A,B,C của tam giác nhọn ABC lần lượt nằm trên các cạnh $B_{1}C_{1},C_{1}A_{1}$ và $A_{1}B_{1}$ của tam giác $A_{1}B_{1}C_{1}$ sao cho $\widehat{ABC}=\widehat{A_{1}B_{1}C_{1}}$, $\widehat{BCA}=\widehat{B_{1}C_{1}A_{1}}$, $\widehat{CAB}=\widehat{C_{1}A_{1}B_{1}}$. Chứng minh rằng hai trực tâm của các tam giác $ABC$ và $A_{1}B_{1}C_{1}$ cách đều tâm đường tròn ngoại tiếp tam giác $ABC$




#708926 [TOPIC] HÌNH HỌC ÔN THI VÀO THPT CHUYÊN 2018-2019

Gửi bởi mduc123 trong 21-05-2018 - 15:48

Bài 77: Cho tam giác ABC nhọn có BC>CA. Gọi O,H lần lượt là tâm đường tròn ngoại tiếp, trực tâm của tam giác ABC. Đường thẳng vuông góc với OF tại F cắt đường thẳng chứa cạnh AC tại P. Chứng minh $\widehat{FHP}=\widehat{BAC}$




#708800 $f(x) = x^n + 29x^{n - 1} + 2009$ với $n \in ,n...

Gửi bởi mduc123 trong 19-05-2018 - 22:27

Lời giải:

Giả sử $f(x)=P(x)Q(x)$, với $P(x)$ và $Q(x)$ là 2 đa thức hệ số nguyên có bậc lớn hơn hoặc bằng 1.

Đặt $P(x)=x^{m}+a_{m-1}x^{m-1}+...+a_{0}$ và $Q(x)=x^{k}+b_{k-1}x^{k-1}+...+b_{0}$ với$m,k\in \mathbb{N}^{*},m+k=n(1\leq m,k\leq n-1)$ và $a_{i},b_{j}\in \mathbb{Z}$ $ (i=\overline{0,m-1},j=\overline{0,k-1})$

Khi đó $a_{0}b_{0}=2009=7^{2}.41$. Suy ra trong 2 số $a_{0}$ và $b_{0}$ có đúng một số chia hết cho 41. Chẳng hạn đó là số $a_{0}$ còn $b_{0}$ không chia hết cho 41. Đồng nhất các hệ số từ đẳng thức $f(x)=P(x)Q(x)$, ta có hệ:

      $\left\{\begin{matrix} a_{0}b_{0}=2009 & & \\ a_{0}b_{1}+a_{1}b_{0}=0 & & \\ a_{0}b_{2}+a_{1}b_{1}+a_{2}b_{0}=0 & & \end{matrix}\right.$

$\Rightarrow \left\{\begin{matrix} a_{1}\vdots 41 & & \\ a_{2}\vdots 41 & & \\ ... & & \end{matrix}\right.$

Nếu r là chỉ số lớn nhất trong các chỉ số i mà $a_{1}\vdots 41$, thì $r\leq m-1$ (để ý là $a_{m}=1$).

Suy ra $r\leq n-2,a_{r}\vdots 41,a_{r+1}\vdots 41$

Ở đây, hệ số của $x^{r+1}$ trong f(x) là  $h_{r+1}=a_{0}b_{r+1}+a_{1}b_{r}+...+a_{r+1}b_{0}$

Rõ ràng $h_{r+1}=a_{0}b_{r+1}+a_{1}b_{r}+...+a_{r+1}b_{0} \vdots 41$ nên $h_{r+1} \neq 0$

Suy ra $m\geq r+1\geq n-1\geq m$ nên $m=n-1$, tức $Q(x)=x+b_{0}$

Do đó $0=f(-b_{0})=(-b_{0})^{n}+9(-b_{0})^{n-1}+2009$ (vô lý, do $(-b_{0})^{n}+9(-b_{0})^{n-1}+2009$ lẻ do $b_{0}$ lẻ )

đpcm




#708694 $f(x) = x^n + 29x^{n - 1} + 2009$ với $n \in ,n...

Gửi bởi mduc123 trong 18-05-2018 - 17:07

Cho hàm số: $f(x) = x^n + 29x^{n - 1} + 2009$ . Chứng minh rằng $f(x)$  không thể phân tích thành tích của 2 đa thức hệ số nguyên có bậc lớn hơn hoặc bằng 1.




#708554 $f(x+f(y))=f(x)+\frac{xf(4y)}{8}+f(f(y)) ,...

Gửi bởi mduc123 trong 16-05-2018 - 20:19

Lời giải: 

P( f(x); y) $\Rightarrow f(f(x)+f(y))=f(f(x))+f(f(y))+\frac{f(x)f(4y)}{8}$

P( f(y); x) $\Rightarrow f(f(y)+f(x))=f(f(y))+f(f(x))+\frac{f(y)f(4x)}{8}$

$\Rightarrow f(x)f(4y)=f(y)f(4x)\Rightarrow \frac{f(4y)}{f(y)}=\frac{f(4x)}{f(x)}=8a$

P( -f(y); y) $\Rightarrow f(0)=0=f(-f(y))-af(y)^{2}+f(f(y))\Rightarrow f(x)+f(-x)=ax^{2}$

Thay x bởi 4x, tính theo 2 cách ta được a=2

$f(x)+f(-x)=2x^{2}\Rightarrow f(x)=x^{2}$

Thử lại:Thỏa mãn

Kết luận




#706873 [TOPIC] SỐ HỌC ÔN TẬP THPT CHUYÊN TOÁN 10 NĂM HỌC 2018-2019

Gửi bởi mduc123 trong 25-04-2018 - 08:29

Bài 107:Cho a,b,c,d là các số tự nhiên sao cho $b^{2}+1=ac,c^{2}+1=bd$. Chứng minh rằng a=3b-c, d=3c-b

Bài 108:Cho p là số nguyên tố, p>3; $n=\frac{2^{2p}-1}{3}$. Chứng minh rằng $2^{n}-2\vdots n$ (có thể dùng định lý Fermat bé)




#706755 [TOPIC] ÔN THI BẤT ĐẲNG THỨC $\boxed{\text{THPT CHUYÊN}}$...

Gửi bởi mduc123 trong 23-04-2018 - 17:57

Bài 83: Cho a,b,c>0 thỏa mãn ab+bc+ca=1.Tìm giá trị lớn nhất của biểu thức:

$P=\frac{a}{\sqrt{1+a^{2}}}+\frac{b}{\sqrt{1+b^{2}}}+\frac{2c}{\sqrt{1+c^{2}}}$




#706749 [TOPIC] SỐ HỌC ÔN TẬP THPT CHUYÊN TOÁN 10 NĂM HỌC 2018-2019

Gửi bởi mduc123 trong 23-04-2018 - 17:11

TOPIC tiếp tục với các bài toán số học hấp dẫn nào:

86) Cho $x$ và $y$ là các số hữu tỷ thỏa mãn:$(x+y)^{3}=xy(3x+3y+2)$. CMR: $\sqrt{1-xy}$ là một số hữu tỷ.

87) Giả sử $a,b$ là các số nguyên dương sao cho $\frac{(a+b)^{2}+a+b}{ab}$ là một số nguyên. Gọi $d$ là một ước chung bất kỳ của $a,b$. CMR: $d\leq \left [ \sqrt{a+b} \right ]$ ( Kí hiệu $\left [ x \right ]$ là số nguyên lớn nhất không vượt quá $x$)

88) Có hay không các số nguyên $x,y,z$ thỏa mãn: $\left | x-2005y \right |+\left | y-2007z \right |+\left | z-2009x \right |=2011^{x}+2013^{y}+2015^{z}$

89) Tìm các số nguyên x,y thỏa mãn: $x^{3}+x^{2}y+xy^{2}+y^{3}=4(x^{2}+y^{2}+xy+3)$

90) Giải phương trình nghiệm tự nhiên: $x^{2}-5x+7=3^{y}$

90.

$x^{2}-5x+7-3^{y}=0\Rightarrow \Delta =4.3^{y}-3=p^{2}$ (do phương trình có nghiệm tự nhiên)

$\Rightarrow p\vdots 3$ 

đặt p=3q $\Rightarrow 4.3^{y}-3=9q^{2}\Leftrightarrow 4.3^{y-1}=3q^{2}+1$ mà $3q^{2}+1$ không chia hết cho 3 nên $4.3^{y-1}$ không chia hết cho 3. Vậy y=1

$\Rightarrow x^{2}-5x+4=0\Rightarrow x\in$ {1;4}




#706723 [TOPIC] ÔN THI BẤT ĐẲNG THỨC $\boxed{\text{THPT CHUYÊN}}$...

Gửi bởi mduc123 trong 23-04-2018 - 12:36

Bài 78(IMO 1984): Cho a, b, c là các số không âm thỏa mãn $a+b+c=1$.CMR:

$0\leq ab+bc+ca-2abc\leq \frac{7}{27}$

Do a+b+c=1 và a,b,c không âm nên $ab+bc+ca-2abc=ab(1-c)+ca(1-b)+bc\geq 0$

Ta có: $ab+bc+ca-2abc=a(b+c)+bc(1-2a)\leq a(1-a)+\frac{(b+c)^{2}}{4}(1-2a)=a(1-a)+\frac{(a-1)^{2}}{4}(1-2a)$ 

Nên ta chỉ cần Cm $a(1-a)+\frac{(a-1)^{2}}{4}(1-2a)\leq \frac{7}{27}$ thật vậy $a(1-a)+\frac{(a-1)^{2}}{4}(1-2a)\leq \frac{7}{27}\Leftrightarrow (a+\frac{1}{6})(a-\frac{1}{3})^{2}\geq 0$ (đpcm)




#706700 Marathon số học Olympic

Gửi bởi mduc123 trong 22-04-2018 - 22:25

Bài 74:Ta có bổ đề : nếu p là ước nguyên tố của x2+y2 thì p có dạng 4k+1

Chứng minh: x2 đồng dư với -y2 ( mod p) 

suy ra -1 là số chính phương mod p

vậy nên p có dạng 4k+1

từ đó nên ta chỉ được giá trị biểu thức đề cho là 1,-1,3,-3

thiếu rồi bạn,mình chỉ ra 1 ví dụ là (x,y)=(14,7) có (14,7)=7=4.1+3 là nghiệm thỏa mãn đề bài

P/s:ta sẽ dùng bổ đề trên nhưng phải phân trường hợp ra, vả lại mình chưa công nhận cách chứng minh bổ đề trên




#706694 [TOPIC] SỐ HỌC ÔN TẬP THPT CHUYÊN TOÁN 10 NĂM HỌC 2018-2019

Gửi bởi mduc123 trong 22-04-2018 - 21:46

Bài 80 tương tự bài 59 :)

Bài 75 : p=q thì giải pt tìm được m rồi suy ra p=q=2 hoặc p=q=5

Giả sử p>q>2 thì (pq,p+q)=1 và (m2+1,m+1)=<2

m=2k-1 thì $\frac{m^2+1}{m+1}=\frac{2k^2-2k+1}{k}$   là  phân  số  tối  giản  nên   pq=2k2-2k+1 và p+q=k . Ta có (p+q)2=k2>pq=2k2-2k+1 là điều vô lí (giải bất phương trình )

Do đó (m2+1,m+1)=1 Cmtt ta cũng có TH này vô lý => kết luận

Bài 77. Nếu p=k với p là snt ta có đpcm

Xét p khác k ta có k2-a2=pk => (k-a)(k+a)=pk

Vì k+a>k, k-a <k nên ta có k=1 và p=(k-a)(k+a)

Vì k-a<k+a nên ta có p=1+a, 1=1-a => p=1 ( vô lí)

=> kết luận
Bài 81 cần sửa đề 

Bài 78. Từ gt ta có (p+1)(p-2)=2a3 với a nguyên

   Ta thấy p=2 thỏa mãn pt  và p=3 không thỏa    . Xét p>3

Vì (p+1) và p-2 nguyên tố cùng nhau nên ta có p+1=2x3, p-2=y3(với x,y nguyên tố cùng nhau và xy=a)=>2x3-y3=3 . Ta cm được x=1,y=-1 là TH duy nhất thỏa mãn => p=-1 (loại) => kết luận

P/S còn hơn 1 tháng nữa là thi rồi ! Chúng ta sắp phải lo ôn thi nên chắc sẽ ít khi lên diễn đàn. Đây là thời khắc cuối cùng để giải cho xong những bài toán

(p+1) và p-2 nguyên tố cùng nhau, cái này chưa đúng rồi,p=5, thì p+1 và p-2 không nguyên tố cùng nhau.

Làm thế này đi gọi $(p+1,p-2)=d\Rightarrow 3\vdots d\Rightarrow d\in {1;3}$

+)Xét d=3, $2a^{3}\vdots 9\Rightarrow a\vdots 27$,đặt p+1=3m, p-2=3m-3,giải tiếp các kiểu

+)Xét d=1,làm như bạn




#706659 [TOPIC] SỐ HỌC ÔN TẬP THPT CHUYÊN TOÁN 10 NĂM HỌC 2018-2019

Gửi bởi mduc123 trong 22-04-2018 - 17:40

Mình xin góp một bài 

Bài 74: (MCMC,1996) Cho ba số nguyên dương khác nhau x,y,z. Chứng minh $(x-y)^{5}+(y-z)^{5}+(z-x)^{5}\vdots 5(x-y)(y-z)(z-x)$




#706649 $1^{k}+2^{k}+...+(p-1)^{k}\equiv 0(mo...

Gửi bởi mduc123 trong 22-04-2018 - 16:54

mỗi số nguyên tố p có ít nhất 1 căn nguyên thủy a nên ta sẽ được {1,2,...,p-1} đồng dư với tập {a,a2,...,ap-1} nên từ đó bạn có thể suy ra 1k+2k+...+(p-1)kđồng dư với ak+(ak)2+...+(ak)p-1

định nghĩa căn nguyên thủy của một số nguyên tố là gì vậy bạn?




#706420 Chứng minh rằng,$\forall S\in N^{*}$ thì tồn tạ...

Gửi bởi mduc123 trong 19-04-2018 - 16:58

Lời giải:

Đặt $S=2^{a}5^{b}k$ với (k,10)=1

Áp dụng định lý Euler ta có:

$10^{\varphi (k)}\equiv 1$ (mod k)

$10^{2\varphi (k)}\equiv 1$ (mod k)

                        ...

$10^{s\varphi (k)}\equiv 1$ (mod k)

$\Rightarrow \sum_{i=1}^{s}10^{i\varphi (k)}\equiv s$ (mod k)

$\Rightarrow A=\sum_{i=1}^{s}10^{i\varphi (k)}\vdots k$

$\Rightarrow \exists P\mid n=10^{P}A\vdots S$

Vậy luôn tồn tại n thỏa mãn đề bài